Đến nội dung

hanguyen445 nội dung

Có 23 mục bởi hanguyen445 (Tìm giới hạn từ 20-05-2020)


Sắp theo                Sắp xếp  

#744601 $n^2 + m$ không là số chính phương với $m | 2n^2$

Đã gửi bởi hanguyen445 on 20-04-2024 - 11:03 trong Số học

Cho $n$ là số nguyên dương và $m$ là ước nguyên dương của $2n^2$ Chứng minh rằng $n^2 + m$ không là số chính phương

Đặt $A=n^2+m$. Giả sử A là một số chính phương, khi đó tồn tại $ B\in N^*$ sao cho $A=B^2$

 

Nếu n=1  thì $m\in \{1;2\}$

+ Xét $m=1$ suy ra $A=n^2+1=B^2\Leftrightarrow (B-n)(B+n)=1\Rightarrow n=0$, mâu thuẫn với n nguyên dương.

+ Nếu $m=2\Rightarrow A=n^2+2=B^2\Leftrightarrow (B-n)(B+n)=2\Rightarrow n=\frac{1}{2}$, mâu thuẫn với n nguyên dương.

Hình gửi kèm

  • Capture.PNG



#744269 với $d$ là số nguyên dương, CMR:không tồn tại các số nguyên $x...

Đã gửi bởi hanguyen445 on 20-03-2024 - 16:28 trong Số học

1/  tìm các số nguyên $x,y,z$ sao cho $x+y+z=0$ và $xy+yz+zx+3=0$.

2/  Cho $t$ là số nguyên, chứng minh rằng nếu tồn tại các số nguyên $x,y,z$ khác $0$ sao cho $x+y+z=0$ và $xy+yz+zx+4t=0$ thì cũng tồn tại các số nguyên $x',y',z'$ khác $0$ sao cho $x'+y'+z'=0$ và $x'y'+y'z'+z'x'+t=0$.

3/ với $d$ là số nguyên dương, chứng minh rằng không tồn tại các số nguyên $x,y,z$ khác $0$ sao cho $x+y+z=0$ và $xy+yz+zx+2^{d}=0$

Bài 2. Ta có

$xy+yz+zx=-4t \Rightarrow  xy+yz+zx\equiv 0(mod 4)$; $x^2+y^2+z^2=-2(xy+yz+zx)\Rightarrow x^2+y^2+z^2\equiv 0(mod 4) (*)$

Chú ý rằng $\forall t\in R $ thì $t^2\equiv 0;1(mod 4)$, kết hợp với (*) ta suy ra $x^2,y^2,z^2\equiv 0(mod 4)\Rightarrow x;y;z\equiv 0(mod 2) $

Đặt $(x;y;z)=(2x';2y';2z')\ne (0;0;0)$ (***), thay ngược lại vào đẳng thức $xy+yz+zx+4t=0$ thì ta có 

$$x'+y'+z'+t=0$$

Từ (***) hiển nhiên $(x';y';z')\ne (0;0;0)$, do đó phép chứng minh hoàn tất.




#744268 với $d$ là số nguyên dương, CMR:không tồn tại các số nguyên $x...

Đã gửi bởi hanguyen445 on 20-03-2024 - 16:19 trong Số học

1/  tìm các số nguyên $x,y,z$ sao cho $x+y+z=0$ và $xy+yz+zx+3=0$.

2/  Cho $t$ là số nguyên, chứng minh rằng nếu tồn tại các số nguyên $x,y,z$ khác $0$ sao cho $x+y+z=0$ và $xy+yz+zx+4t=0$ thì cũng tồn tại các số nguyên $x',y',z'$ khác $0$ sao cho $x'+y'+z'=0$ và $x'y'+y'z'+z'x'+t=0$.

3/ với $d$ là số nguyên dương, chứng minh rằng không tồn tại các số nguyên $x,y,z$ khác $0$ sao cho $x+y+z=0$ và $xy+yz+zx+2^{d}=0$

Bài 1. Do vai trò x, y, z bình đẳng và trong ba số x,y,z luôn có hai số cùng dấu. Không giảm tổng quát giả sử $xy\ge 0$.

Ta có $xy+3=(x+y)^2\Leftrightarrow x^2+y^2+xy=3 (*)$. Nếu trong (*) có ít nhất một số bằng 0, hiển nhiên vô lý. Do đó $x^2+y^2+xy\ge 3$, 

như vậy (*) xảy ra khi và chỉ khi (x;y) nhận các cặp là $(1;1); (-1;-1)$, thay ngược lại tương ứng z nhận {-2;2}.

Vậy bộ (x;y;z) cần tìm là (1;1;-2) và (-1;-1;2)




#742880 ​$\frac{a}{b(b+2c)^2}+\frac{b}...

Đã gửi bởi hanguyen445 on 03-01-2024 - 15:03 trong Bất đẳng thức và cực trị

Cho $a; b; c$ là các số thực dương thoả mãn $(a+b)(b+c)(c+a)=1$. Chứng minh rằng: 
$\frac{a}{b(b+2c)^2}+\frac{b}{c(c+2a)^2}+\frac{c}{a(a+2b)^2}\geq \frac{4}{3}$

Ta có $ \frac{a}{b(b+2c)^2}=\frac{(\frac{a}{b+2c})^2}{ab}$. Biến đổi tương tự với các số hạng còn lại trong biểu thức vế trái và sử dụng BĐT cauchy-schwarz ta có $$VT\ge (\frac{a}{b+2c}+\frac{b}{c+2a}+\frac{c}{a+2b})^2.\frac{1}{ab+bc+ac}$$

Lại có $\frac{a}{b+2c}+\frac{b}{c+2a}+\frac{c}{a+2b}\ge \frac{(a+b+c)^2}{3(ab+bc+ac)}$. Khi đó

$$VT\ge\frac{(a+b+c)^4}{9(ab+bc+ac)^3}\ge\frac{1}{ab+bc+ac}$$

Có $1=(a+b)(b+c)(c+a)=(a+b+c)(ab+bc+ac)-abc\ge\frac{8}{9}. (ab+bc+ac)(a+b+c)\ge\frac{8}{9}(ab+bc+ac)\sqrt{3(ab+bc+ac)}$.

Hay $ab+bc+ac\le\frac{3}{4}$. Do đó ta suy ra được $VT\ge\frac{4}{3}$. Phép chứng minh hoàn tất.




#742561 $\frac{xy}{z}+\frac{yz}{x...

Đã gửi bởi hanguyen445 on 18-12-2023 - 15:30 trong Bất đẳng thức và cực trị

BÀI TOÁN 8. Với $a, b, c,  m, n, p, x, y, z$ là các số thực dương chứng minh rằng $$(a^3+b^3+c^3) (m^3+n^3+p^3) (x^3+y^3+z^3) \geq (amx+bny+cpz)^3$$

Đây là Bất đẳng thức Holder cho 3 dãy. Diễn đàn đã có chứng minh tại đây, tuy nhiên thì bạn nào chưa biết bất đẳng thức này có thể thử tự tìm cách giải. Gợi ý: sử dụng AM-GM. 

 

BÀI TOÁN 9. Với $a, b, c$ là các số thực dương chứng minh rằng 

b) $P=\frac{a}{\sqrt{2b^2+2c^2-a^2}}+\frac{b}{\sqrt{2a^2+2c^2-b^2}}+\frac{c}{\sqrt{2a^2+2b^2-c^2}} \geq \sqrt{3}$

Bài toán 9b: Sử dụng BĐT cauchy-schwarz ta có

$P=\sum\frac{a^2}{\sqrt{a(2ab^2+2ac^2-a^3)}}\ge\frac{(a+b+c)^2}{\sqrt{(a+b+c)T}}$ với $T=\sum (2ab^2+2ac^2-a^3)$

Lại có $T=\sum 2(a+b+c)(ab+bc+ac)-a^3-b^3-c^3-6abc$ và sử dụng BĐT schur ta có $ a^3+b^3+c^3+3abc\ge\sum ab(a+b)$

Do đó suy ra $T\le 2(a+b+c)(ab+bc+ac)-(a+b+c)(ab+bc+ac)=(a+b+c)(ab+bc+ac)\le\frac{(a+b+c)^3}{3}$.

Như vậy thì $P\ge \sqrt{3}$. Phép chứng minh hoàn tất.

Chú ý: Để bất đẳng thức xảy ra thì ta cần thêm điều kiện $Min (b^2+c^2-a^2/2; a^2+c^2-b^2/2; a^2+b^2-c^2/2)>0$




#742560 $\frac{xy}{z}+\frac{yz}{x...

Đã gửi bởi hanguyen445 on 18-12-2023 - 15:04 trong Bất đẳng thức và cực trị

BÀI TOÁN 8. Với $a, b, c,  m, n, p, x, y, z$ là các số thực dương chứng minh rằng $$(a^3+b^3+c^3) (m^3+n^3+p^3) (x^3+y^3+z^3) \geq (amx+bny+cpz)^3$$

Đây là Bất đẳng thức Holder cho 3 dãy. Diễn đàn đã có chứng minh tại đây, tuy nhiên thì bạn nào chưa biết bất đẳng thức này có thể thử tự tìm cách giải. Gợi ý: sử dụng AM-GM. 

 

BÀI TOÁN 9. Với $a, b, c$ là các số thực dương chứng minh rằng 

a) $\frac{a}{\sqrt{a^2+8bc}}+\frac{b}{\sqrt{b^2+8ac}}+\frac{c}{\sqrt{c^2+8ab}} \geq 1$

b) $\frac{a}{\sqrt{2b^2+2c^2-a^2}}+\frac{b}{\sqrt{2a^2+2c^2-b^2}}+\frac{c}{\sqrt{2a^2+2b^2-c^2}} \geq \sqrt{3}$

Bài toán 9 a: 

Đặt $P=\sum\frac{a}{\sqrt{a^2+8bc}}$. Sử BĐT cauchy-schwarz ta có

$$P=\sum \frac{a^2}{a\sqrt{a^2+8bc}}\ge\frac{(a+b+c)^2}{\sqrt{a(a^3+8abc)}}\ge\frac{(a+b+c)^2}{\sqrt{(a+b+c)(\sum(a^3+8abc)}}$$

Lại có

 

$\sum a(a^2+8bc)=a^3+b^3+c^3+24abc$

 

$=(a+b+c)(a^2+b^2+c^2-ab-bc-ac)+27abc$

 

$\le (a+b+c)(a^2+b^2+c^2-ab-bc-ac)+3(a+b+c)(ab+bc+ac)=(a+b+c)^3$.

 

Do đó ta suy ra được $P\ge\frac{(a+b+c)^2}{(a+b+c)^2}=1$. Hoàn tất chứng minh




#742514 $\frac{xy}{z}+\frac{yz}{x...

Đã gửi bởi hanguyen445 on 15-12-2023 - 00:18 trong Bất đẳng thức và cực trị

BÀI TOÁN 7. Tìm giá trị nhỏ nhất của biểu thức $$S=\frac{3a}{b+c}+\frac{4b}{a+c}+\frac{5c}{a+b}.$$

Biến đổi S và kết hợp với sử dụng BĐT Cauchy-Shwarz ta có

$S+12=(a+b+c)(\frac{3}{b+c}+\frac{4}{a+c}+\frac{5}{a+b})$

$\geq (a+b+c)\frac{(2+\sqrt{3}+\sqrt{5})^2}{2(a+b+c)}=\frac{(2+\sqrt{3}+\sqrt{5})^2}{2}$ 

Hay $S\ge\frac{(2+\sqrt{3}+\sqrt{5})^2}{2}-12$. Do đó $Min S =\frac{(2+\sqrt{3}+\sqrt{5})^2}{2}-12$. 




#742486 Chứng minh bất đẳng thức $\sum\frac{1}{a^2+2bc...

Đã gửi bởi hanguyen445 on 13-12-2023 - 00:13 trong Bất đẳng thức và cực trị

Bài toán 1: Cho các số thực dương $a,b,c$ thỏa mãn $a+b+c=1$. Chứng minh rằng 

$$\frac{1}{a^2+2bc}+\frac{1}{b^2+2ac}+\frac{1}{c^2+2ab}\ge\frac{1}{\sqrt{a^2-ab+b^2}}+\frac{1}{\sqrt{b^2-bc+c^2}}+\frac{1}{\sqrt{c^2-ca+a^2}}$$

 

Bài toán 2: Cho các số thực dương $a,b,c$ thỏa mãn $abc=1$. Chứng minh rằng

$$\frac{a^3+1}{\sqrt{a^4+b+c}}+\frac{b^3+1}{\sqrt{b^4+a+c}}+\frac{c^3+1}{\sqrt{c^4+a+b}}\ge 2\sqrt{ab+bc+ac}$$




#742469 $\frac{xy}{z}+\frac{yz}{x...

Đã gửi bởi hanguyen445 on 11-12-2023 - 17:11 trong Bất đẳng thức và cực trị

 

Thấy chỗ này sôi nổi quá nên cho mình ké bài của mình ạ:vv
Chứng minh bất đẳng thức sau $$\frac{a}{(b+2c)^2\sqrt{b^2+3}}+\frac{b}{(c+2a)^2\sqrt{c^2+3}}+\frac{c}{(a+2b)^2\sqrt{a^2+3}}\geq\frac{1}{2(a+b+c)}$$
2) Với $a,b,c$ là các số thực dương và $ab+bc+ca=3$
 

 

Trường hợp số 2: Biến đổi  và kết hợp với sử dụng BĐT cauchy-schwarz ta có
 
$$VT=\sum (\frac{a}{b+2c})^2.\frac{1}{a\sqrt{b^2+3}}\ge (\sum\frac{a}{b+2c})^2.\frac{1}{\sum a\sqrt{b^2+3}}$$
 
 
$$\sum\frac{a}{b+2c}=\sum\frac{a^2}{ab+2ac}\ge\frac{(a+b+c)^2}{3(ab+bc+ac)}=\frac{3}{ab+bc+ac}$$
 
$$\sum a\sqrt{b^2+3}\le\sqrt{(a^2+b^2+c^2)(a^2+b^2+c^2+9)}$$
 
Do đó ta suy ra được $VT\ge\frac{(a+b+c)^4}{(ab+bc+ac)^2 \sqrt{(a^2+b^2+c^2)(a^2+b^2+c^2+9)}}$. 
Đặt $M=(ab+bc+ac)^3(a^2+b^2+c^2)(a^2+b^2+c^2+9)$, hơn nữa ta lại có:
 
$$4^3(ab+bc+ac)^3.4(a^2+b^2+c^2)(a^2+b^2+c^2+9)\le (\frac{12(ab+bc+ac)+5(a^2+b^2+c^2+9)}{5})^5$$
 
Có  $12(ab+bc+ac)+5(a^2+b^2+c^2)+9\le \dfrac{20}{3}(a+b+c)^2$. Suy ra $M\le\frac{4}{3^5}(a+b+c)^{10}$
Suy ra $ (ab+bc+ac)^2\sqrt{(a^2+b^2+c^2)(a^2+b^2+c^2+9)}=\sqrt{(ab+bc+ac).M}\le\frac{2}{9}(a+b+c)^5$
 
Do đó ta suy ra được $VT\ge\frac{(a+b+c)^4}{2(a+b+c)^5}$. Hay $VT\ge\frac{1}{2(a+b+c)}$.
Như vậy phép chứng minh hoàn tất.



#742468 $\frac{xy}{z}+\frac{yz}{x...

Đã gửi bởi hanguyen445 on 11-12-2023 - 16:39 trong Bất đẳng thức và cực trị

 

Thấy chỗ này sôi nổi quá nên cho mình ké bài của mình ạ:vv
Chứng minh bất đẳng thức sau $$\frac{a}{(b+2c)^2\sqrt{b^2+3}}+\frac{b}{(c+2a)^2\sqrt{c^2+3}}+\frac{c}{(a+2b)^2\sqrt{a^2+3}}\geq\frac{1}{2(a+b+c)}$$
3) Với $a,b,c$ là các số thực dương và $a+b+c=3$

 

Trường hợp số 3: Biến đổi  và kết hợp với sử dụng BĐT cauchy-schwarz ta có

$$VT=\sum (\frac{a}{b+2c})^2.\frac{1}{a\sqrt{b^2+3}}\ge (\sum\frac{a}{b+2c})^2.\frac{1}{\sum a\sqrt{b^2+3}}$$

$$\sum\frac{a}{b+2c}=\sum\frac{a^2}{ab+2ac}\ge\frac{(a+b+c)^2}{3(ab+bc+ac)}=\frac{3}{ab+bc+ac}$$

$$\sum a\sqrt{b^2+3}\le\sqrt{(a^2+b^2+c^2)(a^2+b^2+c^2+9)}$$

Do đó ta suy ra được $VT\ge\frac{9}{(ab+bc+ac)^2 \sqrt{(a^2+b^2+c^2)(a^2+b^2+c^2+9)}}$. Hơn nữa ta lại có

$$M=4^4(ab+bc+ac)^4.4(a^2+b^2+c^2)(a^2+b^2+c^2+9)\le (\frac{16(ab+bc+ac)+5(a^2+b^2+c^2+9)}{6})^6$$

Ta lại có $16(ab+bc+ac)+5(a^2+b^2+c^2)+9\le 7(a+b+c)^2+9=72$.

Suy ra $(ab+bc+ac)^2 \sqrt{(a^2+b^2+c^2)(a^2+b^2+c^2+9)}\le 54$.

 

Như vậy thì $VT\ge\frac{9}{54}=\frac{1}{6}=\frac{1}{2(a+b+c)}$. Do đó phép chứng minh hoàn tất.




#742454 $\frac{xy}{z}+\frac{yz}{x...

Đã gửi bởi hanguyen445 on 10-12-2023 - 15:41 trong Bất đẳng thức và cực trị

BÀI TOÁN 6. Giả sử $x, y, z \geq 1$ và $\frac{1}{x}+\frac{1}{y}+\frac{1}{z}=2$. Chứng minh $$\sqrt{x+y+z} \geq \sqrt{x-1}+\sqrt{y-1}+\sqrt{z-1}.$$      

Biến đổi và sử dụng BĐT cauchy-schwarz ta có  $\frac{1}{x}=2\Leftrightarrow \sum\frac{x-1}{x}=1\ge\dfrac{ (\sum\sqrt{x-1})^2}{x+y+z}$

Hay $\sqrt{x+y+z}\ge\sum\sqrt{x-1}$. Do đó phép chứng minh hoàn tất.




#742335 $\frac{a+b+c}{3}-\sqrt[3]{abc}...

Đã gửi bởi hanguyen445 on 02-12-2023 - 23:28 trong Bất đẳng thức và cực trị

cho a,b,c là các số thực dương. CMR:
$\frac{a+b+c}{3}-\sqrt[3]{abc}\leq max {(\sqrt{a}-\sqrt{b})^{2},(\sqrt{b}-\sqrt{c})^{2},(\sqrt{c}-\sqrt{a})^{2}}$

                                                                                    (đề chọn đội tuyển dự thi Toán quốc tế, Mỹ năm 2000)

Do bài toán đối xứng với ba biến $a, b, c$, không giảm tổng quát giả sử $a\ge b\ge c$.

Lúc này ta suy ra được $(\sqrt{a}-\sqrt{c})^2=\text{Max}( (\sqrt{a}-\sqrt{c})^2;(\sqrt{a}-\sqrt{b})^2;(\sqrt{b}-\sqrt{c})^2)$

Do đó BĐT cần chứng minh tương đương

 

$$\frac{a+b+c}{3}-\sqrt[3]{abc}\le (\sqrt{c}-\sqrt{a})^{2}$$

 

Đặt $(x;y;z)=(\sqrt[6]{a},\sqrt[6]{b},\sqrt[6]{c})$ và hiển nhiên $x\ge y\ge z\ge 0$,  khi đó bất đẳng thức viết lại thành 

$$x^6+y^6+z^6-3(xyz)^2\le 3(x^3-z^3)^2$$

Ta có:

 

$x^6+y^6+z^6-3(xyz)^2=(x^2+y^2+z^2)[(x^2-z^2)^2+(y^2-x^2)(y^2-z^2)$

 

$3(x^3-z^3)^2=3(x-z)^2(x^2+xz+z^2)^2$

 

Hiển nhiên $(y^2-x^2)(y^2-z^2)\le 0$ với mọi $x\ge y\ge z$. Như vậy bất đẳng thức đúng nếu ta chứng minh được

$$(x^2+y^2+z^2)(x+z)^2\le 3(x^2+xz+z^2)^2(**)$$

Ta có $(x^2+xz+z^2)\ge\frac{3}{4} (x+z)^2$. Do đó ta suy ra được $VP(**)\ge\frac{9}{4}(x+z)^2(x^2+xz+z^2)$, nên BĐT đúng nếu

$$9(x^2+xz+z^2)\ge 4(x^2+y^2+z^2) (3*)$$

Hiển nhiên: $9x^2>8x^2>4(x^2+y^2)$ và $9(xz+z^2)\ge 4z^2$. Cộng vế ta suy ra được (3*) được chứng minh.

 

Vậy phép chứng minh hoàn tất.




#742322 $\frac{xy}{z}+\frac{yz}{x...

Đã gửi bởi hanguyen445 on 02-12-2023 - 10:04 trong Bất đẳng thức và cực trị

 

Thấy chỗ này sôi nổi quá nên cho mình ké bài của mình ạ:vv
Chứng minh bất đẳng thức sau $$\frac{a}{(b+2c)^2\sqrt{b^2+3}}+\frac{b}{(c+2a)^2\sqrt{c^2+3}}+\frac{c}{(a+2b)^2\sqrt{a^2+3}}\geq\frac{1}{2(a+b+c)}$$
1) Với $a,b,c$ là các số thực dương và $a^2+b^2+c^2=3$
2) Với $a,b,c$ là các số thực dương và $ab+bc+ca=3$
3) Với $a,b,c$ là các số thực dương và $a+b+c=3$

 

Trường hợp 1:

Ta có $\frac{a}{(b+2c)^2\sqrt{b^2+3}}=\dfrac{(\dfrac{a}{b+2c})^2}{a\sqrt{b^2+3}}$, biến đổi tương tự với các số hạng còn lại và sử dụng BĐT cauchy-schwarz cho vế trái. Ta có:

$VT\ge P^2.\frac{1}{Q}$ với:

$$P=\frac{a}{b+2c}+\frac{b}{c+2a}+\frac{c}{a+2b}\ge \frac{(a+b+c)^2}{3(ab+bc+ac)}\ge 1 (**)$$

$$Q=a\sqrt{b^2+3}+b\sqrt{c^2+3}+c\sqrt{a^2+3}=\sum\sqrt{a}\sqrt{ab^2+3a}\le \sqrt{(a+b+c)(ab^2+bc^2+ca^2+3(a+b+c))}$$

Ta lại có $ a^3+ac^2+a^2c\ge 3ac^2$, lập luận tương tự và cộng vế ta có:

$ 3(ab^2+bc^2+ca^2)\le (a+b+c)(a^2+b^2+c^2)=3(a+b+c)\Leftrightarrow ab^2+bc^2+ca^2\le a+b+c$, hay $Q\le 2(a+b+c) (***)$

Từ (**) và (***), ta suy ra được $VT\ge\dfrac{1}{2(a+b+c)}$. Do đó phép chứng minh hoàn tất.




#742277 $\frac{xy}{z}+\frac{yz}{x...

Đã gửi bởi hanguyen445 on 28-11-2023 - 20:10 trong Bất đẳng thức và cực trị

BÀI TOÁN 4.

Cho các số thực dương $a, b, c$. Chứng minh rằng $$ \frac{a}{(b+c)^2}+\frac{b}{(c+a)^2}+\frac{c}{(a+b)^2} \geq \frac{9}{4(a+b+c)}.$$

Biến đổi, kết hợp với sử dụng BĐT cauchy-schwarz, ta có $$\sum\frac{a}{(b+c)^2}=\sum\dfrac{(\dfrac{a}{b+c})^2}{a}\ge\frac{1}{a+b+c}(\sum\frac{a}{b+c})^2 (1)$$

Hơn nữa ta có $$\sum\frac{a}{b+c}=\sum\frac{a^2}{ab+ac}\ge\frac{(a+b+c)^2}{2(ab+bc+ac)}\ge\frac{3(ab+bc+ac)}{2(ab+bc+ac)} (2)$$

Từ (1) và (2) ta suy ra được $\sum\frac{a}{(b+c)^2}\ge\frac{9}{a+b+c}$. Phép chứng minh hoàn tất




#742260 Bài toán bất đẳng thức hình học tam giác $\triangle ABC$ và tứ...

Đã gửi bởi hanguyen445 on 27-11-2023 - 11:42 trong Hình học

Bài toán 1: Chứng minh rằng trong một tam giác, tổng độ dài của hai cạnh luôn lớn hơn cạnh còn lại.




#742259 $\frac{xy}{z}+\frac{yz}{x...

Đã gửi bởi hanguyen445 on 27-11-2023 - 11:35 trong Bất đẳng thức và cực trị

Bài toán 3. Cho các số dương $a, b, c$ thoả mãn điều kiện $a+b+c=3$. Chứng minh bất đẳng thức $$\frac{a}{1+b^2}+\frac{b}{1+c^2}+\frac{c}{1+a^2} \geq \frac{3}{2}.$$

Ta có $\frac{a}{1+b^2}=\frac{a(1+b^2)-ab^2}{1+b^2}=a-\dfrac{ab^2}{1+b^2}\ge a-\dfrac{ab}{2}$. Lập luận tương tự và cộng vế ta có

$$VT\ge (a+b+c)-\dfrac{ab+bc+ac}{2}\ge 3-\dfrac{(a+b+c)^2}{6}=\frac{3}{2}$$

Phép chứng minh hoàn tất.




#742245 $\frac{xy}{z}+\frac{yz}{x...

Đã gửi bởi hanguyen445 on 26-11-2023 - 21:41 trong Bất đẳng thức và cực trị

Bài toán 2. Chứng minh rằng với mọi $a, b, c, d$ dương ta luôn có $$16(abc+bcd+cda+dab) \leq (a+b+c+d)^3.$$

 

Ta có $16(abc+bcd+cda+dab) =16bc(a+d)+16ad(b+c)\leq 4(b+c)^2(a+d)+4(b+c)(a+d)^2=4(a+d)(b+c)(a+b+c+d)$

Lại có $4(a+d)(b+c)\le (a+b+c+d)^2$

 

Do đó ta suy ra được $16(abc+bcd+cda+dab) \le (a+b+c+d)^3$. 

Vậy phép chứng minh hoàn tất. 




#742235 Chứng minh rằng diện tích của hình chữ nhật $MNPQ$ nhỏ hơn 6

Đã gửi bởi hanguyen445 on 26-11-2023 - 15:24 trong Hình học

Cho tam giác ABC có $AB=4$, $BC=6$. Trên cạnh AB lấy điểm M, cạnh AC lấy điểm N và BC lấy các điểm P, Q sao cho tứ giác $MNPQ$ là hình chữ nhật. Chứng minh rằng diện tích của hình chữ nhật $MNPQ$ nhỏ hơn 6.




#742233 $\frac{xy}{z}+\frac{yz}{x...

Đã gửi bởi hanguyen445 on 26-11-2023 - 14:55 trong Bất đẳng thức và cực trị

Cho các số thực dương $x, y, z$ thoả mãn $x^2+y^2+z^2=3$. Chứng minh $\frac{xy}{z}+\frac{yz}{x}+\frac{zx}{y} \geq 3$. 

Bất đẳng thức cần chứng minh biến đổi tương đương thành:

$\sum(xy)^2\ge 3xyz\Leftrightarrow \sum(xy)^2\ge xyz\sqrt{3(x^2+y^2+z^2)}\Leftrightarrow\ (\sum (xy)^2)^2\ge 3x^2y^2z^2(x^2+y^2+z^2)$

 

Đặt $(x^2;y^2;z^2)=(a;b;c)\geq (0;0;0)$. Khi đó BĐT cần chứng minh viết lại thành:

$(ab+bc+ac)^2\ge 3abc(a+b+c)\Leftrightarrow\sum (ab-bc)^2\ge 0$  (*). Hiển nhiên (*) luôn đúng. Do đó ta có điều phải chứng minh.




#742207 $ \frac{(b+c-a)^2}{b^2+c^2+bc} + \frac...

Đã gửi bởi hanguyen445 on 24-11-2023 - 16:46 trong Bất đẳng thức và cực trị

Cho a,b,c>0.CMR: $ \frac{(b+c-a)^2}{b^2+c^2+bc} + \frac{(c+a-b)^2}{c^2+a^2+ca} + \frac{(a+b-c)^2}{a^2+b^2+ab} \ge 1$

Đặt $P=\sum\frac{(b+c-a)^2}{b^2+c^2+bc}$

 

Sử dụng BĐT cauchy-schwarz , ta có đánh giá:

$P+1=P+\frac{a+b+c}{a+b+c}=\sum (\frac{(b+c-a)^2}{b^2+c^2+bc}+\frac{a}{a+b+c})=\sum (\frac{(b+c-a)^2}{b^2+c^2+bc}+\frac{a^2}{a^2+ab+ac})\ge\sum\frac{(b+c)^2}{a^2+b^2+c^2+ab+bc+ac})=Q\\$

Hay $P+1\ge\ Q$ với \[Q = \frac{{{{\left( {b + c} \right)}^2}}}{{{a^2} + {b^2} + {c^2} + ab + bc + ac}} + \frac{{{{\left( {a + b} \right)}^2}}}{{{a^2} + {b^2} + {c^2} + ab + bc + ac}} + \frac{{{{\left( {a + c} \right)}^2}}}{{{a^2} + {b^2} + {c^2} + ab + bc + ac}} = 2\]

Do đó ta có $P+1\ge 2\Leftrightarrow P\ge 1$. Hoàn tất chứng minh.



#731083 Với các số thực $a, b, c$ thỏa mãn $a^2 + b^2 + c^2 = 3$,...

Đã gửi bởi hanguyen445 on 10-10-2021 - 15:00 trong Bất đẳng thức và cực trị

Với các số thực $a, b, c$ thỏa mãn $a^2 + b^2 + c^2 = 3$, tìm giá trị nhỏ nhất của biểu thức $P = \sum{|2a^3 + bc|}$.

Cách 1: Xét BĐT phụ $|a+b|\ge |a|-|b|$  (1). Dấu = xảy ra khi và chỉ khi ab<=0 và |a|>=|b|.  

Sử dụng BĐT (1) ta suy ra được $P\ge 2\sum|a^3|-\sum |b||c|$=2(x^3+y^3+z^3)-(xy+yz+zx)$ với $x^2+y^2+z^2=3; x,y,z>=0$

Ta có $x^2+y^2+z^2=3>=xy+yz+zx$; $2\sum x^3=\sum (x^3+x^3+1)-3\ge 3(x^2+y^2+z^2)-3=9-3=6$

Suy ra được $P\ge 6-3=3$. Dấu = xảy ra khi (a,b,c)=(1,1,-1), (-1,-1,-1)

Cách 2: Xét dấu

Do bất đẳng thức đối xứng với ba biến a, b, c nên không giảm tổng quát ta giả sử $a\ge b\ge c$

TH1. Nếu a, b,c cùng không âm thì hiển nhiên ta có $P=|2a^3+bc|+|2b^3+ac|+|2c^3+ab|=2\sum |a^3|+\sum |b||c|$. Do đó  trong trường

hợp này luôn có thể giả sử $a\ge b\ge c\ge 0$. Nên P=$2(a^3+b^3+c^3)+(ab+bc+ac)$

TH2. Nếu $a\ge 0\ge b\ge c$ thì $P=2a^3+(-b)(-c)-2b^3+a(-c)-2c^3+a(-b)$.

Do đó trong trường hợp này thay $(a,b,c) -> (a,-b,-c)$ với a,b,c>=0 thì ta qui về bài toán trường hợp 1.

Lúc này bài toán của TH1,TH2 là: Tìm GTNN P=2\sum a^3+\sum ab với a,b,c không âm thỏa mãn: $a^2+b^2+c^2=3$. Thật vậy:

Ta có  $P=\sum (a^3+a^3+1)+\sum ab-3\ge  3(a^2+b^2+c^2)-3=3.3-3=6$

 

 

TH3. Nếu $a\ge b\ge 0\ge c$. Khi đó ta thay (a,b,c) bởi $(a,b,-c) với a,b,c>=0$. Lúc này tìm GTNN của biểu thức 

$P=|2a^3-bc|+|2b^3-ac|+|ab-2c^3|\ge 2(a^3+b^3+c^3)-(ab+bc+ac)=\sum (a^3+a^3+1)-\sum ab-3\ge 3\sum a^2+\sum ab-3$

Từ $a^2+b^2+c^2=3\ge ab+bc+ac$. Suy ra được $P\ge 3.3-3-3=3$

Dấu "=" xảy ra khi và chỉ khi (a,b,-c)=(1,1,1) hay (a,b,c) ban đầu bằng (1,1,-1) và các hoán vị.

 

TH4. Nếu a,b,c cùng âm thì thay (a,b,c) bởi (-a;-b;-c), sao cho a,b,c>=0 thì ta có:

$P=\sum |-2a^3+bc|\ge \sum (2a^3-bc)=2(a^3+b^3+c^3)-(ab+bc+ac)>=3 $ tương tự trường hợp 3. 

Dấu = xảy ra khi và chỉ khi $a=b=c=-1$

 

Như vậy giá trị nhỏ nhất của P bằng 3 khi (a,b,c)=(1,1,-1) và các hoán vị và (a,b,c)=(-1,-1,-1)




#730850 Tìm tất cả hàm số $f:\mathbb{R}\rightarrow \mat...

Đã gửi bởi hanguyen445 on 02-10-2021 - 10:02 trong Phương trình hàm

Tìm tất cả hàm số $f:\mathbb{R}\rightarrow \mathbb{R}$ thỏa mãn:

$f(x)=max_{y\in \mathbb{R}}\left \{ xy-f(y) \right \}\forall x\in \mathbb{R}$

Với mỗi $x\in \mathbb(R)$, hàm số $g(y)=xy-f(y)$ đạt giá trị lớn nhất trên $\mathbb(R)$, nên tồn tại $y_0\in \mathbb(R)$ sao cho $Max g(y)=g(y_0)=xy_0-f(y_0)$.

Suy ra $f(x)=xy_0-f(y_0)$ hay $ f(x)=ax+b$ với $a, b$ là các hằng số cho trước.

Và thay trở lại hàm $g(y)$ ta có: $g(y)=xy-ay-b=y(x-a)-b$, thay x bởi a+1 thì ta có $g(y)=y-b$, hiển nhiên hàm số này không có giá trị

lớn nhất trên $R$.

Do vậy không tồn tại hàm số thỏa mãn yêu cầu bài toán. (bạn xem giúp không biết có đúng không).




#729534 Bài toán tìm bộ số nguyên ($a$, $b$, $c$) thỏa...

Đã gửi bởi hanguyen445 on 09-08-2021 - 16:06 trong Số học

Bài toán 1: Tìm tất cả các bộ số nguyên (a,b,c) thỏa mãn $a^2+ab+b^2=5c^2$

 

Bài toán 2: Cho hai số nguyên dương $a$, $b$. Gọi $q$ và $r$ lần lượt là thương và dư trong phép chia $ab$ cho $a+b$  sao cho $q^2+r=2269$. Tìm tất cả các giá trị có thể có của $|a-b|$.